Aquí sólo discutiremos el caso de representaciones irreducibles finito-dimensionales (irreps) de un álgebra de Lie compleja semisimple $L$ .
Recordemos que el conjunto $Z$ de invariantes de Casimir es la centro $Z(U(L))$ de la álgebra envolvente universal $U(L)$ cf. por ejemplo este Correo de Phys.SE.
La pregunta de OP se responde sin pruebas en la p. 253 de la Ref. 1:
Teorema 2. Para toda álgebra de Lie semisimple $L$ de rango $r$ existe un conjunto de $r$ polinomio invariante del generador $t_a$ cuyos valores propios caracterizan las representaciones irreducibles de dimensión finita.
La Ref. 2 (que es uno de los libros más importantes sobre álgebras de Lie, al menos si uno está interesado en las pruebas) no se molesta en mencionar el Teorema 2 explícitamente. Sin embargo, es posible encadenar un conjunto de resultados más fundamentales (y sus pruebas) de la Ref. 2 para obtener el resultado buscado. A continuación esbozamos la estrategia de demostración.
Recordemos además que hay asociada una sistema radicular $\Phi$ al álgebra de Lie $L$ e imaginemos que hemos elegido una base $\Delta$ para $\Phi$ . La orden $|W|$ de la Grupo de Weyl $W$ es igual a las posibles elecciones de bases (desordenadas) e igual a las posibles elecciones de bases (fundamentales) Cámaras de Weyl .
Se demuestra en los capítulos 20-21 de la Ref. 2. que un irrep de dimensión finita tiene un único vector de mayor peso (único hasta la normalización) con alguna dominante integral peso $\lambda$ . A partir de ahora denotaremos tales irrep $V(\lambda)$ . (La Ref. 2. también define una noción de irrep $V(\lambda)$ cuando $\lambda$ es integral pero no dominante. Tales irreps son necesariamente de dimensión infinita, así que las ignoraremos). De ello se deduce que
Dos irreps $V(\lambda)$ y $V(\mu)$ son equivalentes (es decir, isomorfas) si sus pesos más altos son iguales $\lambda=\mu$ .
Como consecuencia de Teorema de Harish-Chandra el conjunto $Z$ de Casimires toma el mismo valor en dos irreps de mayor peso $V(\lambda)$ y $V(\mu)$ si $\lambda+\delta$ y $\mu+\delta$ pertenecen a la misma órbita de Weyl,
$$ \sigma(\lambda+\delta)~=~\mu+\delta, \qquad \sigma \in W. $$
Aquí $\delta$ es la mitad de la suma de las raíces positivas. Sin embargo, si ambos pesos integrales $\lambda$ y $\mu$ son dominantes, entonces $\lambda+\delta$ y $\mu+\delta$ deben pertenecer ambas a (el interior de) la cámara fundamental de Weyl, de modo que la reflexión de Weyl $\sigma={\bf 1}$ debe ser el elemento de identidad. En conclusión, obtenemos que
El conjunto $Z$ de Casimires toma el mismo valor en dos irreps de dimensión finita $V(\lambda)$ y $V(\mu)$ si sus pesos más altos son iguales $\lambda=\mu$ .
El teorema de Harish-Chandra se demuestra en el capítulo 23 de la Ref. 2. Véase también este y este entradas relacionadas con Math.SE.
Ejemplo: Consideremos el álgebra de Lie $L=sl(3,\mathbb{C})$ . El grupo de Weyl es $S_3$ . El álgebra de Lie $L$ tiene dos invariantes de Casimir independientes $C_2$ y $C_3$ ,
$$C_n ~:=~ {\rm str}({\rm ad} t_{a_1}\circ\ldots\circ{\rm ad} t_{a_n}) t^{a_1} \otimes\ldots\otimes t^{a_n}, \qquad n~\in~ \{2,3\}.$$
Consideremos la representación fundamental tridimensional $F$ y el dual/contragrediente representación $\bar{F}$ de $L$ que son irreps no equivalentes. Tienen pesos máximos $\lambda=(1,0)$ y $\mu=(0,1)$ respectivamente. En concreto, si $t_a$ , $a=1, \ldots, 8$ son generadores de $L=sl(3,\mathbb{C})$ (consejo de sombrero: Peter Kravchuk)
$$\bar{F}(t_a)~=~ -F(t_a)^t,$$
para que los Casimires $C_2$ (y $C_3$ ) toman el mismo valor (opuesto) en $F$ y $\bar{F}$
$$ {\rm tr}_{\bar{F}}\bar{F}(C_n)~=~(-1)^n{\rm tr}_{F}F(C_n), \qquad n~\in~ \{2,3\}. $$
Se puede demostrar que los valores son distintos de cero, de modo que las Casimires $C_2$ y $C_3$ distinguir entre los dos irreps no equivalentes $F$ y $\bar{F}$ como debe ser.
Referencias:
-
A. O. Barut y R. Raczka, Teoría de representaciones de grupos y aplicaciones, 2ª ed., 1980.
-
J.E. Humphreys, Introducción a las álgebras de Lie y a la teoría de la representación, (1980).
0 votos
La misma pregunta en Math.SE: math.stackexchange.com/q/288816/11127